$mathfrak p$ be a prime ideal of a commutative ring $R$ , then $R_mathfrak p/mathfrak pR_mathfrak p$ is the...












5












$begingroup$


Let $mathfrak p$ be a prime ideal of a commutative ring $R$ with unity , then how to show that the field $R_mathfrak p/mathfrak pR_mathfrak p$ is isomorphic with the field of fractions of the integral domain $R/mathfrak p$ ?



I can show that $R_mathfrak p/mathfrak pR_mathfrak p cong (R/mathfrak p)_mathfrak p$ as $R_mathfrak p$ modules ... but I am not sure whether this implies the result I am asking or not










share|cite|improve this question











$endgroup$








  • 1




    $begingroup$
    Does this help? math.stackexchange.com/questions/502448/…
    $endgroup$
    – Paul K
    Sep 4 '17 at 15:34










  • $begingroup$
    @PaulK : Not as it is ... please see my edit
    $endgroup$
    – user
    Sep 4 '17 at 15:39










  • $begingroup$
    $(R/P)_P$ is the field of fractions of $R/P$!
    $endgroup$
    – Bernard
    Sep 4 '17 at 15:42






  • 1




    $begingroup$
    @Bernard : True .. but the isomorphism I get is an $R_P$ module isomorphism not a ring isomorphism ...
    $endgroup$
    – user
    Sep 4 '17 at 15:44






  • 1




    $begingroup$
    It's also a ring homomorphism.
    $endgroup$
    – Bernard
    Sep 4 '17 at 16:02
















5












$begingroup$


Let $mathfrak p$ be a prime ideal of a commutative ring $R$ with unity , then how to show that the field $R_mathfrak p/mathfrak pR_mathfrak p$ is isomorphic with the field of fractions of the integral domain $R/mathfrak p$ ?



I can show that $R_mathfrak p/mathfrak pR_mathfrak p cong (R/mathfrak p)_mathfrak p$ as $R_mathfrak p$ modules ... but I am not sure whether this implies the result I am asking or not










share|cite|improve this question











$endgroup$








  • 1




    $begingroup$
    Does this help? math.stackexchange.com/questions/502448/…
    $endgroup$
    – Paul K
    Sep 4 '17 at 15:34










  • $begingroup$
    @PaulK : Not as it is ... please see my edit
    $endgroup$
    – user
    Sep 4 '17 at 15:39










  • $begingroup$
    $(R/P)_P$ is the field of fractions of $R/P$!
    $endgroup$
    – Bernard
    Sep 4 '17 at 15:42






  • 1




    $begingroup$
    @Bernard : True .. but the isomorphism I get is an $R_P$ module isomorphism not a ring isomorphism ...
    $endgroup$
    – user
    Sep 4 '17 at 15:44






  • 1




    $begingroup$
    It's also a ring homomorphism.
    $endgroup$
    – Bernard
    Sep 4 '17 at 16:02














5












5








5


1



$begingroup$


Let $mathfrak p$ be a prime ideal of a commutative ring $R$ with unity , then how to show that the field $R_mathfrak p/mathfrak pR_mathfrak p$ is isomorphic with the field of fractions of the integral domain $R/mathfrak p$ ?



I can show that $R_mathfrak p/mathfrak pR_mathfrak p cong (R/mathfrak p)_mathfrak p$ as $R_mathfrak p$ modules ... but I am not sure whether this implies the result I am asking or not










share|cite|improve this question











$endgroup$




Let $mathfrak p$ be a prime ideal of a commutative ring $R$ with unity , then how to show that the field $R_mathfrak p/mathfrak pR_mathfrak p$ is isomorphic with the field of fractions of the integral domain $R/mathfrak p$ ?



I can show that $R_mathfrak p/mathfrak pR_mathfrak p cong (R/mathfrak p)_mathfrak p$ as $R_mathfrak p$ modules ... but I am not sure whether this implies the result I am asking or not







abstract-algebra ring-theory commutative-algebra maximal-and-prime-ideals localization






share|cite|improve this question















share|cite|improve this question













share|cite|improve this question




share|cite|improve this question








edited Jan 26 at 7:18









Andrews

1,2691421




1,2691421










asked Sep 4 '17 at 15:24









useruser

1




1








  • 1




    $begingroup$
    Does this help? math.stackexchange.com/questions/502448/…
    $endgroup$
    – Paul K
    Sep 4 '17 at 15:34










  • $begingroup$
    @PaulK : Not as it is ... please see my edit
    $endgroup$
    – user
    Sep 4 '17 at 15:39










  • $begingroup$
    $(R/P)_P$ is the field of fractions of $R/P$!
    $endgroup$
    – Bernard
    Sep 4 '17 at 15:42






  • 1




    $begingroup$
    @Bernard : True .. but the isomorphism I get is an $R_P$ module isomorphism not a ring isomorphism ...
    $endgroup$
    – user
    Sep 4 '17 at 15:44






  • 1




    $begingroup$
    It's also a ring homomorphism.
    $endgroup$
    – Bernard
    Sep 4 '17 at 16:02














  • 1




    $begingroup$
    Does this help? math.stackexchange.com/questions/502448/…
    $endgroup$
    – Paul K
    Sep 4 '17 at 15:34










  • $begingroup$
    @PaulK : Not as it is ... please see my edit
    $endgroup$
    – user
    Sep 4 '17 at 15:39










  • $begingroup$
    $(R/P)_P$ is the field of fractions of $R/P$!
    $endgroup$
    – Bernard
    Sep 4 '17 at 15:42






  • 1




    $begingroup$
    @Bernard : True .. but the isomorphism I get is an $R_P$ module isomorphism not a ring isomorphism ...
    $endgroup$
    – user
    Sep 4 '17 at 15:44






  • 1




    $begingroup$
    It's also a ring homomorphism.
    $endgroup$
    – Bernard
    Sep 4 '17 at 16:02








1




1




$begingroup$
Does this help? math.stackexchange.com/questions/502448/…
$endgroup$
– Paul K
Sep 4 '17 at 15:34




$begingroup$
Does this help? math.stackexchange.com/questions/502448/…
$endgroup$
– Paul K
Sep 4 '17 at 15:34












$begingroup$
@PaulK : Not as it is ... please see my edit
$endgroup$
– user
Sep 4 '17 at 15:39




$begingroup$
@PaulK : Not as it is ... please see my edit
$endgroup$
– user
Sep 4 '17 at 15:39












$begingroup$
$(R/P)_P$ is the field of fractions of $R/P$!
$endgroup$
– Bernard
Sep 4 '17 at 15:42




$begingroup$
$(R/P)_P$ is the field of fractions of $R/P$!
$endgroup$
– Bernard
Sep 4 '17 at 15:42




1




1




$begingroup$
@Bernard : True .. but the isomorphism I get is an $R_P$ module isomorphism not a ring isomorphism ...
$endgroup$
– user
Sep 4 '17 at 15:44




$begingroup$
@Bernard : True .. but the isomorphism I get is an $R_P$ module isomorphism not a ring isomorphism ...
$endgroup$
– user
Sep 4 '17 at 15:44




1




1




$begingroup$
It's also a ring homomorphism.
$endgroup$
– Bernard
Sep 4 '17 at 16:02




$begingroup$
It's also a ring homomorphism.
$endgroup$
– Bernard
Sep 4 '17 at 16:02










2 Answers
2






active

oldest

votes


















5












$begingroup$

There's an obvious map $fcolon R_Pto text{Frac}(R/P)$, given by $a/bmapsto (a+P)/(b+P)$. You need to check:




  • If $a/bin R_P$, then $f(a/b)in text{Frac}(R/P)$, i.e., $b+Pneq 0$ in $R/P$.

  • If $a/b = c/d$ in $R_P$, then $f(a/b) = f(c/d)$.

  • $f$ is a homomorphism.


To show that $f$ induces an isomorphism $R_P/PR_Pcong text{Frac}(R/P)$, you just need to show that the kernel of $f$ is $PR_P$. Well, suppose $a/b$ is in the kernel. Then $f(a/b) = (a+P)/(b+P)$ is equal to $0$ in $text{Frac}(R/P)$, so $a+P$ is equal to $0$ in $R/P$. So $ain P$, and $a/b = a(1/b) in PR_P$. Conversely, if $a/bin PR_P$, then it is equivalent to $p/b'$ for some $pin P$ and $b'notin P$. And $f(p/b') = (p+P)/(b'+P) = 0$, so $a/b$ is in the kernel (you've already checked that $f$ is well-defined, so it suffices to look at just one representative of its equivalence class).



There's a more category-theoretic way to do this exercise, where you check that $R_P/PR_P$ and $text{Frac}(R/P)$ both satisfy the universal property that a ring homomorphism from this ring to $S$ is uniquely determined by a ring homomorphism $Rto S$ which maps every element of $P$ to $0$ and every element of $Rbackslash P$ to a unit, and hence they must be isomorphic (by Yoneda's Lemma). But it sounds like you might benefit more at this stage from working through the details of the more "concrete" proof.






share|cite|improve this answer











$endgroup$









  • 1




    $begingroup$
    +1 I also think that OP will be benefited from "working through the details of the more "concrete" proof.".
    $endgroup$
    – Krish
    Sep 4 '17 at 15:53










  • $begingroup$
    @Alex Kruckman : why $a=pa'$ ?
    $endgroup$
    – user
    Sep 4 '17 at 16:25










  • $begingroup$
    @users Because that's how you multiply in the localization! If $a/b = (p/1)(a'/b')$, then $a = pa'$ and $b = 1b'$.
    $endgroup$
    – Alex Kruckman
    Sep 4 '17 at 17:32












  • $begingroup$
    @AlexKruckman : But $a/b=pa'/b'$ implies $s(b' a - bpa')=0 $ for some $s notin P$ ...
    $endgroup$
    – user
    Sep 4 '17 at 17:35










  • $begingroup$
    So $sb'a = sbpa'in P$. Since $snotin P$, and $b'notin P$, and $P$ is prime, $ain P$. But ok, maybe I've been a little sloppy in what I wrote above, I"ll rewrite that paragraph.
    $endgroup$
    – Alex Kruckman
    Sep 4 '17 at 17:44



















2












$begingroup$

Since localization preserves exact sequences we have that the exact sequence

$$ 0 to mathfrak p to R to R/mathfrak p to 0$$
becomes

$$0 to mathfrak p R_{mathfrak p} to R_{mathfrak p} to (R/mathfrak p)_{mathfrak p} to 0$$
Hence $(R/mathfrak p)_{mathfrak p}=R_{mathfrak p}/mathfrak pR_mathfrak p$. From this you can conclude that it is the field of fractions since $R/mathfrak p$ is a domain and localizing by $mathfrak p$ means inverting all nonzero elements (there are no nonzero divisor in an integral domain).






share|cite|improve this answer











$endgroup$









  • 2




    $begingroup$
    this proof is better
    $endgroup$
    – Sky
    Sep 4 '17 at 15:47










  • $begingroup$
    Better of what?
    $endgroup$
    – SC30
    Sep 4 '17 at 15:47






  • 2




    $begingroup$
    This proof gives an isomorphism of modules - is there a way to see automatically that it gives an isomorphism of rings as well?
    $endgroup$
    – Alex Kruckman
    Sep 4 '17 at 15:48










  • $begingroup$
    @SC30 but you still need to explain $(R/p)_p$ is isomorphic to the fraction field!better we know this localization.
    $endgroup$
    – Sky
    Sep 4 '17 at 15:51










  • $begingroup$
    @SC30 : Like Alex Kruckman , I am also tempted to comment that this only shows module isomorphism , which I already know as stated in the question
    $endgroup$
    – user
    Sep 4 '17 at 15:51











Your Answer





StackExchange.ifUsing("editor", function () {
return StackExchange.using("mathjaxEditing", function () {
StackExchange.MarkdownEditor.creationCallbacks.add(function (editor, postfix) {
StackExchange.mathjaxEditing.prepareWmdForMathJax(editor, postfix, [["$", "$"], ["\\(","\\)"]]);
});
});
}, "mathjax-editing");

StackExchange.ready(function() {
var channelOptions = {
tags: "".split(" "),
id: "69"
};
initTagRenderer("".split(" "), "".split(" "), channelOptions);

StackExchange.using("externalEditor", function() {
// Have to fire editor after snippets, if snippets enabled
if (StackExchange.settings.snippets.snippetsEnabled) {
StackExchange.using("snippets", function() {
createEditor();
});
}
else {
createEditor();
}
});

function createEditor() {
StackExchange.prepareEditor({
heartbeatType: 'answer',
autoActivateHeartbeat: false,
convertImagesToLinks: true,
noModals: true,
showLowRepImageUploadWarning: true,
reputationToPostImages: 10,
bindNavPrevention: true,
postfix: "",
imageUploader: {
brandingHtml: "Powered by u003ca class="icon-imgur-white" href="https://imgur.com/"u003eu003c/au003e",
contentPolicyHtml: "User contributions licensed under u003ca href="https://creativecommons.org/licenses/by-sa/3.0/"u003ecc by-sa 3.0 with attribution requiredu003c/au003e u003ca href="https://stackoverflow.com/legal/content-policy"u003e(content policy)u003c/au003e",
allowUrls: true
},
noCode: true, onDemand: true,
discardSelector: ".discard-answer"
,immediatelyShowMarkdownHelp:true
});


}
});














draft saved

draft discarded


















StackExchange.ready(
function () {
StackExchange.openid.initPostLogin('.new-post-login', 'https%3a%2f%2fmath.stackexchange.com%2fquestions%2f2416524%2fmathfrak-p-be-a-prime-ideal-of-a-commutative-ring-r-then-r-mathfrak-p%23new-answer', 'question_page');
}
);

Post as a guest















Required, but never shown

























2 Answers
2






active

oldest

votes








2 Answers
2






active

oldest

votes









active

oldest

votes






active

oldest

votes









5












$begingroup$

There's an obvious map $fcolon R_Pto text{Frac}(R/P)$, given by $a/bmapsto (a+P)/(b+P)$. You need to check:




  • If $a/bin R_P$, then $f(a/b)in text{Frac}(R/P)$, i.e., $b+Pneq 0$ in $R/P$.

  • If $a/b = c/d$ in $R_P$, then $f(a/b) = f(c/d)$.

  • $f$ is a homomorphism.


To show that $f$ induces an isomorphism $R_P/PR_Pcong text{Frac}(R/P)$, you just need to show that the kernel of $f$ is $PR_P$. Well, suppose $a/b$ is in the kernel. Then $f(a/b) = (a+P)/(b+P)$ is equal to $0$ in $text{Frac}(R/P)$, so $a+P$ is equal to $0$ in $R/P$. So $ain P$, and $a/b = a(1/b) in PR_P$. Conversely, if $a/bin PR_P$, then it is equivalent to $p/b'$ for some $pin P$ and $b'notin P$. And $f(p/b') = (p+P)/(b'+P) = 0$, so $a/b$ is in the kernel (you've already checked that $f$ is well-defined, so it suffices to look at just one representative of its equivalence class).



There's a more category-theoretic way to do this exercise, where you check that $R_P/PR_P$ and $text{Frac}(R/P)$ both satisfy the universal property that a ring homomorphism from this ring to $S$ is uniquely determined by a ring homomorphism $Rto S$ which maps every element of $P$ to $0$ and every element of $Rbackslash P$ to a unit, and hence they must be isomorphic (by Yoneda's Lemma). But it sounds like you might benefit more at this stage from working through the details of the more "concrete" proof.






share|cite|improve this answer











$endgroup$









  • 1




    $begingroup$
    +1 I also think that OP will be benefited from "working through the details of the more "concrete" proof.".
    $endgroup$
    – Krish
    Sep 4 '17 at 15:53










  • $begingroup$
    @Alex Kruckman : why $a=pa'$ ?
    $endgroup$
    – user
    Sep 4 '17 at 16:25










  • $begingroup$
    @users Because that's how you multiply in the localization! If $a/b = (p/1)(a'/b')$, then $a = pa'$ and $b = 1b'$.
    $endgroup$
    – Alex Kruckman
    Sep 4 '17 at 17:32












  • $begingroup$
    @AlexKruckman : But $a/b=pa'/b'$ implies $s(b' a - bpa')=0 $ for some $s notin P$ ...
    $endgroup$
    – user
    Sep 4 '17 at 17:35










  • $begingroup$
    So $sb'a = sbpa'in P$. Since $snotin P$, and $b'notin P$, and $P$ is prime, $ain P$. But ok, maybe I've been a little sloppy in what I wrote above, I"ll rewrite that paragraph.
    $endgroup$
    – Alex Kruckman
    Sep 4 '17 at 17:44
















5












$begingroup$

There's an obvious map $fcolon R_Pto text{Frac}(R/P)$, given by $a/bmapsto (a+P)/(b+P)$. You need to check:




  • If $a/bin R_P$, then $f(a/b)in text{Frac}(R/P)$, i.e., $b+Pneq 0$ in $R/P$.

  • If $a/b = c/d$ in $R_P$, then $f(a/b) = f(c/d)$.

  • $f$ is a homomorphism.


To show that $f$ induces an isomorphism $R_P/PR_Pcong text{Frac}(R/P)$, you just need to show that the kernel of $f$ is $PR_P$. Well, suppose $a/b$ is in the kernel. Then $f(a/b) = (a+P)/(b+P)$ is equal to $0$ in $text{Frac}(R/P)$, so $a+P$ is equal to $0$ in $R/P$. So $ain P$, and $a/b = a(1/b) in PR_P$. Conversely, if $a/bin PR_P$, then it is equivalent to $p/b'$ for some $pin P$ and $b'notin P$. And $f(p/b') = (p+P)/(b'+P) = 0$, so $a/b$ is in the kernel (you've already checked that $f$ is well-defined, so it suffices to look at just one representative of its equivalence class).



There's a more category-theoretic way to do this exercise, where you check that $R_P/PR_P$ and $text{Frac}(R/P)$ both satisfy the universal property that a ring homomorphism from this ring to $S$ is uniquely determined by a ring homomorphism $Rto S$ which maps every element of $P$ to $0$ and every element of $Rbackslash P$ to a unit, and hence they must be isomorphic (by Yoneda's Lemma). But it sounds like you might benefit more at this stage from working through the details of the more "concrete" proof.






share|cite|improve this answer











$endgroup$









  • 1




    $begingroup$
    +1 I also think that OP will be benefited from "working through the details of the more "concrete" proof.".
    $endgroup$
    – Krish
    Sep 4 '17 at 15:53










  • $begingroup$
    @Alex Kruckman : why $a=pa'$ ?
    $endgroup$
    – user
    Sep 4 '17 at 16:25










  • $begingroup$
    @users Because that's how you multiply in the localization! If $a/b = (p/1)(a'/b')$, then $a = pa'$ and $b = 1b'$.
    $endgroup$
    – Alex Kruckman
    Sep 4 '17 at 17:32












  • $begingroup$
    @AlexKruckman : But $a/b=pa'/b'$ implies $s(b' a - bpa')=0 $ for some $s notin P$ ...
    $endgroup$
    – user
    Sep 4 '17 at 17:35










  • $begingroup$
    So $sb'a = sbpa'in P$. Since $snotin P$, and $b'notin P$, and $P$ is prime, $ain P$. But ok, maybe I've been a little sloppy in what I wrote above, I"ll rewrite that paragraph.
    $endgroup$
    – Alex Kruckman
    Sep 4 '17 at 17:44














5












5








5





$begingroup$

There's an obvious map $fcolon R_Pto text{Frac}(R/P)$, given by $a/bmapsto (a+P)/(b+P)$. You need to check:




  • If $a/bin R_P$, then $f(a/b)in text{Frac}(R/P)$, i.e., $b+Pneq 0$ in $R/P$.

  • If $a/b = c/d$ in $R_P$, then $f(a/b) = f(c/d)$.

  • $f$ is a homomorphism.


To show that $f$ induces an isomorphism $R_P/PR_Pcong text{Frac}(R/P)$, you just need to show that the kernel of $f$ is $PR_P$. Well, suppose $a/b$ is in the kernel. Then $f(a/b) = (a+P)/(b+P)$ is equal to $0$ in $text{Frac}(R/P)$, so $a+P$ is equal to $0$ in $R/P$. So $ain P$, and $a/b = a(1/b) in PR_P$. Conversely, if $a/bin PR_P$, then it is equivalent to $p/b'$ for some $pin P$ and $b'notin P$. And $f(p/b') = (p+P)/(b'+P) = 0$, so $a/b$ is in the kernel (you've already checked that $f$ is well-defined, so it suffices to look at just one representative of its equivalence class).



There's a more category-theoretic way to do this exercise, where you check that $R_P/PR_P$ and $text{Frac}(R/P)$ both satisfy the universal property that a ring homomorphism from this ring to $S$ is uniquely determined by a ring homomorphism $Rto S$ which maps every element of $P$ to $0$ and every element of $Rbackslash P$ to a unit, and hence they must be isomorphic (by Yoneda's Lemma). But it sounds like you might benefit more at this stage from working through the details of the more "concrete" proof.






share|cite|improve this answer











$endgroup$



There's an obvious map $fcolon R_Pto text{Frac}(R/P)$, given by $a/bmapsto (a+P)/(b+P)$. You need to check:




  • If $a/bin R_P$, then $f(a/b)in text{Frac}(R/P)$, i.e., $b+Pneq 0$ in $R/P$.

  • If $a/b = c/d$ in $R_P$, then $f(a/b) = f(c/d)$.

  • $f$ is a homomorphism.


To show that $f$ induces an isomorphism $R_P/PR_Pcong text{Frac}(R/P)$, you just need to show that the kernel of $f$ is $PR_P$. Well, suppose $a/b$ is in the kernel. Then $f(a/b) = (a+P)/(b+P)$ is equal to $0$ in $text{Frac}(R/P)$, so $a+P$ is equal to $0$ in $R/P$. So $ain P$, and $a/b = a(1/b) in PR_P$. Conversely, if $a/bin PR_P$, then it is equivalent to $p/b'$ for some $pin P$ and $b'notin P$. And $f(p/b') = (p+P)/(b'+P) = 0$, so $a/b$ is in the kernel (you've already checked that $f$ is well-defined, so it suffices to look at just one representative of its equivalence class).



There's a more category-theoretic way to do this exercise, where you check that $R_P/PR_P$ and $text{Frac}(R/P)$ both satisfy the universal property that a ring homomorphism from this ring to $S$ is uniquely determined by a ring homomorphism $Rto S$ which maps every element of $P$ to $0$ and every element of $Rbackslash P$ to a unit, and hence they must be isomorphic (by Yoneda's Lemma). But it sounds like you might benefit more at this stage from working through the details of the more "concrete" proof.







share|cite|improve this answer














share|cite|improve this answer



share|cite|improve this answer








edited Sep 4 '17 at 17:46

























answered Sep 4 '17 at 15:42









Alex KruckmanAlex Kruckman

28.1k32658




28.1k32658








  • 1




    $begingroup$
    +1 I also think that OP will be benefited from "working through the details of the more "concrete" proof.".
    $endgroup$
    – Krish
    Sep 4 '17 at 15:53










  • $begingroup$
    @Alex Kruckman : why $a=pa'$ ?
    $endgroup$
    – user
    Sep 4 '17 at 16:25










  • $begingroup$
    @users Because that's how you multiply in the localization! If $a/b = (p/1)(a'/b')$, then $a = pa'$ and $b = 1b'$.
    $endgroup$
    – Alex Kruckman
    Sep 4 '17 at 17:32












  • $begingroup$
    @AlexKruckman : But $a/b=pa'/b'$ implies $s(b' a - bpa')=0 $ for some $s notin P$ ...
    $endgroup$
    – user
    Sep 4 '17 at 17:35










  • $begingroup$
    So $sb'a = sbpa'in P$. Since $snotin P$, and $b'notin P$, and $P$ is prime, $ain P$. But ok, maybe I've been a little sloppy in what I wrote above, I"ll rewrite that paragraph.
    $endgroup$
    – Alex Kruckman
    Sep 4 '17 at 17:44














  • 1




    $begingroup$
    +1 I also think that OP will be benefited from "working through the details of the more "concrete" proof.".
    $endgroup$
    – Krish
    Sep 4 '17 at 15:53










  • $begingroup$
    @Alex Kruckman : why $a=pa'$ ?
    $endgroup$
    – user
    Sep 4 '17 at 16:25










  • $begingroup$
    @users Because that's how you multiply in the localization! If $a/b = (p/1)(a'/b')$, then $a = pa'$ and $b = 1b'$.
    $endgroup$
    – Alex Kruckman
    Sep 4 '17 at 17:32












  • $begingroup$
    @AlexKruckman : But $a/b=pa'/b'$ implies $s(b' a - bpa')=0 $ for some $s notin P$ ...
    $endgroup$
    – user
    Sep 4 '17 at 17:35










  • $begingroup$
    So $sb'a = sbpa'in P$. Since $snotin P$, and $b'notin P$, and $P$ is prime, $ain P$. But ok, maybe I've been a little sloppy in what I wrote above, I"ll rewrite that paragraph.
    $endgroup$
    – Alex Kruckman
    Sep 4 '17 at 17:44








1




1




$begingroup$
+1 I also think that OP will be benefited from "working through the details of the more "concrete" proof.".
$endgroup$
– Krish
Sep 4 '17 at 15:53




$begingroup$
+1 I also think that OP will be benefited from "working through the details of the more "concrete" proof.".
$endgroup$
– Krish
Sep 4 '17 at 15:53












$begingroup$
@Alex Kruckman : why $a=pa'$ ?
$endgroup$
– user
Sep 4 '17 at 16:25




$begingroup$
@Alex Kruckman : why $a=pa'$ ?
$endgroup$
– user
Sep 4 '17 at 16:25












$begingroup$
@users Because that's how you multiply in the localization! If $a/b = (p/1)(a'/b')$, then $a = pa'$ and $b = 1b'$.
$endgroup$
– Alex Kruckman
Sep 4 '17 at 17:32






$begingroup$
@users Because that's how you multiply in the localization! If $a/b = (p/1)(a'/b')$, then $a = pa'$ and $b = 1b'$.
$endgroup$
– Alex Kruckman
Sep 4 '17 at 17:32














$begingroup$
@AlexKruckman : But $a/b=pa'/b'$ implies $s(b' a - bpa')=0 $ for some $s notin P$ ...
$endgroup$
– user
Sep 4 '17 at 17:35




$begingroup$
@AlexKruckman : But $a/b=pa'/b'$ implies $s(b' a - bpa')=0 $ for some $s notin P$ ...
$endgroup$
– user
Sep 4 '17 at 17:35












$begingroup$
So $sb'a = sbpa'in P$. Since $snotin P$, and $b'notin P$, and $P$ is prime, $ain P$. But ok, maybe I've been a little sloppy in what I wrote above, I"ll rewrite that paragraph.
$endgroup$
– Alex Kruckman
Sep 4 '17 at 17:44




$begingroup$
So $sb'a = sbpa'in P$. Since $snotin P$, and $b'notin P$, and $P$ is prime, $ain P$. But ok, maybe I've been a little sloppy in what I wrote above, I"ll rewrite that paragraph.
$endgroup$
– Alex Kruckman
Sep 4 '17 at 17:44











2












$begingroup$

Since localization preserves exact sequences we have that the exact sequence

$$ 0 to mathfrak p to R to R/mathfrak p to 0$$
becomes

$$0 to mathfrak p R_{mathfrak p} to R_{mathfrak p} to (R/mathfrak p)_{mathfrak p} to 0$$
Hence $(R/mathfrak p)_{mathfrak p}=R_{mathfrak p}/mathfrak pR_mathfrak p$. From this you can conclude that it is the field of fractions since $R/mathfrak p$ is a domain and localizing by $mathfrak p$ means inverting all nonzero elements (there are no nonzero divisor in an integral domain).






share|cite|improve this answer











$endgroup$









  • 2




    $begingroup$
    this proof is better
    $endgroup$
    – Sky
    Sep 4 '17 at 15:47










  • $begingroup$
    Better of what?
    $endgroup$
    – SC30
    Sep 4 '17 at 15:47






  • 2




    $begingroup$
    This proof gives an isomorphism of modules - is there a way to see automatically that it gives an isomorphism of rings as well?
    $endgroup$
    – Alex Kruckman
    Sep 4 '17 at 15:48










  • $begingroup$
    @SC30 but you still need to explain $(R/p)_p$ is isomorphic to the fraction field!better we know this localization.
    $endgroup$
    – Sky
    Sep 4 '17 at 15:51










  • $begingroup$
    @SC30 : Like Alex Kruckman , I am also tempted to comment that this only shows module isomorphism , which I already know as stated in the question
    $endgroup$
    – user
    Sep 4 '17 at 15:51
















2












$begingroup$

Since localization preserves exact sequences we have that the exact sequence

$$ 0 to mathfrak p to R to R/mathfrak p to 0$$
becomes

$$0 to mathfrak p R_{mathfrak p} to R_{mathfrak p} to (R/mathfrak p)_{mathfrak p} to 0$$
Hence $(R/mathfrak p)_{mathfrak p}=R_{mathfrak p}/mathfrak pR_mathfrak p$. From this you can conclude that it is the field of fractions since $R/mathfrak p$ is a domain and localizing by $mathfrak p$ means inverting all nonzero elements (there are no nonzero divisor in an integral domain).






share|cite|improve this answer











$endgroup$









  • 2




    $begingroup$
    this proof is better
    $endgroup$
    – Sky
    Sep 4 '17 at 15:47










  • $begingroup$
    Better of what?
    $endgroup$
    – SC30
    Sep 4 '17 at 15:47






  • 2




    $begingroup$
    This proof gives an isomorphism of modules - is there a way to see automatically that it gives an isomorphism of rings as well?
    $endgroup$
    – Alex Kruckman
    Sep 4 '17 at 15:48










  • $begingroup$
    @SC30 but you still need to explain $(R/p)_p$ is isomorphic to the fraction field!better we know this localization.
    $endgroup$
    – Sky
    Sep 4 '17 at 15:51










  • $begingroup$
    @SC30 : Like Alex Kruckman , I am also tempted to comment that this only shows module isomorphism , which I already know as stated in the question
    $endgroup$
    – user
    Sep 4 '17 at 15:51














2












2








2





$begingroup$

Since localization preserves exact sequences we have that the exact sequence

$$ 0 to mathfrak p to R to R/mathfrak p to 0$$
becomes

$$0 to mathfrak p R_{mathfrak p} to R_{mathfrak p} to (R/mathfrak p)_{mathfrak p} to 0$$
Hence $(R/mathfrak p)_{mathfrak p}=R_{mathfrak p}/mathfrak pR_mathfrak p$. From this you can conclude that it is the field of fractions since $R/mathfrak p$ is a domain and localizing by $mathfrak p$ means inverting all nonzero elements (there are no nonzero divisor in an integral domain).






share|cite|improve this answer











$endgroup$



Since localization preserves exact sequences we have that the exact sequence

$$ 0 to mathfrak p to R to R/mathfrak p to 0$$
becomes

$$0 to mathfrak p R_{mathfrak p} to R_{mathfrak p} to (R/mathfrak p)_{mathfrak p} to 0$$
Hence $(R/mathfrak p)_{mathfrak p}=R_{mathfrak p}/mathfrak pR_mathfrak p$. From this you can conclude that it is the field of fractions since $R/mathfrak p$ is a domain and localizing by $mathfrak p$ means inverting all nonzero elements (there are no nonzero divisor in an integral domain).







share|cite|improve this answer














share|cite|improve this answer



share|cite|improve this answer








edited Sep 4 '17 at 15:50

























answered Sep 4 '17 at 15:45









SC30SC30

43028




43028








  • 2




    $begingroup$
    this proof is better
    $endgroup$
    – Sky
    Sep 4 '17 at 15:47










  • $begingroup$
    Better of what?
    $endgroup$
    – SC30
    Sep 4 '17 at 15:47






  • 2




    $begingroup$
    This proof gives an isomorphism of modules - is there a way to see automatically that it gives an isomorphism of rings as well?
    $endgroup$
    – Alex Kruckman
    Sep 4 '17 at 15:48










  • $begingroup$
    @SC30 but you still need to explain $(R/p)_p$ is isomorphic to the fraction field!better we know this localization.
    $endgroup$
    – Sky
    Sep 4 '17 at 15:51










  • $begingroup$
    @SC30 : Like Alex Kruckman , I am also tempted to comment that this only shows module isomorphism , which I already know as stated in the question
    $endgroup$
    – user
    Sep 4 '17 at 15:51














  • 2




    $begingroup$
    this proof is better
    $endgroup$
    – Sky
    Sep 4 '17 at 15:47










  • $begingroup$
    Better of what?
    $endgroup$
    – SC30
    Sep 4 '17 at 15:47






  • 2




    $begingroup$
    This proof gives an isomorphism of modules - is there a way to see automatically that it gives an isomorphism of rings as well?
    $endgroup$
    – Alex Kruckman
    Sep 4 '17 at 15:48










  • $begingroup$
    @SC30 but you still need to explain $(R/p)_p$ is isomorphic to the fraction field!better we know this localization.
    $endgroup$
    – Sky
    Sep 4 '17 at 15:51










  • $begingroup$
    @SC30 : Like Alex Kruckman , I am also tempted to comment that this only shows module isomorphism , which I already know as stated in the question
    $endgroup$
    – user
    Sep 4 '17 at 15:51








2




2




$begingroup$
this proof is better
$endgroup$
– Sky
Sep 4 '17 at 15:47




$begingroup$
this proof is better
$endgroup$
– Sky
Sep 4 '17 at 15:47












$begingroup$
Better of what?
$endgroup$
– SC30
Sep 4 '17 at 15:47




$begingroup$
Better of what?
$endgroup$
– SC30
Sep 4 '17 at 15:47




2




2




$begingroup$
This proof gives an isomorphism of modules - is there a way to see automatically that it gives an isomorphism of rings as well?
$endgroup$
– Alex Kruckman
Sep 4 '17 at 15:48




$begingroup$
This proof gives an isomorphism of modules - is there a way to see automatically that it gives an isomorphism of rings as well?
$endgroup$
– Alex Kruckman
Sep 4 '17 at 15:48












$begingroup$
@SC30 but you still need to explain $(R/p)_p$ is isomorphic to the fraction field!better we know this localization.
$endgroup$
– Sky
Sep 4 '17 at 15:51




$begingroup$
@SC30 but you still need to explain $(R/p)_p$ is isomorphic to the fraction field!better we know this localization.
$endgroup$
– Sky
Sep 4 '17 at 15:51












$begingroup$
@SC30 : Like Alex Kruckman , I am also tempted to comment that this only shows module isomorphism , which I already know as stated in the question
$endgroup$
– user
Sep 4 '17 at 15:51




$begingroup$
@SC30 : Like Alex Kruckman , I am also tempted to comment that this only shows module isomorphism , which I already know as stated in the question
$endgroup$
– user
Sep 4 '17 at 15:51


















draft saved

draft discarded




















































Thanks for contributing an answer to Mathematics Stack Exchange!


  • Please be sure to answer the question. Provide details and share your research!

But avoid



  • Asking for help, clarification, or responding to other answers.

  • Making statements based on opinion; back them up with references or personal experience.


Use MathJax to format equations. MathJax reference.


To learn more, see our tips on writing great answers.




draft saved


draft discarded














StackExchange.ready(
function () {
StackExchange.openid.initPostLogin('.new-post-login', 'https%3a%2f%2fmath.stackexchange.com%2fquestions%2f2416524%2fmathfrak-p-be-a-prime-ideal-of-a-commutative-ring-r-then-r-mathfrak-p%23new-answer', 'question_page');
}
);

Post as a guest















Required, but never shown





















































Required, but never shown














Required, but never shown












Required, but never shown







Required, but never shown

































Required, but never shown














Required, but never shown












Required, but never shown







Required, but never shown







Popular posts from this blog

Can a sorcerer learn a 5th-level spell early by creating spell slots using the Font of Magic feature?

Does disintegrating a polymorphed enemy still kill it after the 2018 errata?

A Topological Invariant for $pi_3(U(n))$